Proving two subgroups with same cardinality are identical if one is normal











up vote
2
down vote

favorite












I need to prove the following statement:



Let $G$ be a finite group, $H, K subset G$ two subgroups, $gcd(Card(H), Card(G/H))=1, Card(H)=Card(K), H$ normal in G. Then $H=K.$



We know that Card($G$) = Card($H$) [G:H], so Card($G$) = Card($H$) Card($G/H$). According to one of the isomorphism theorems, $KH<G, (Kcap H) triangleleft H $, and $f: K rightarrow G/H$ is a group homomorphism with ker $ f= Kcap H$ and $K/(K cap H) cong KH/H.$



Somehow I can not connect the dots. Can somebody help me.
Many thanks.










share|cite|improve this question


























    up vote
    2
    down vote

    favorite












    I need to prove the following statement:



    Let $G$ be a finite group, $H, K subset G$ two subgroups, $gcd(Card(H), Card(G/H))=1, Card(H)=Card(K), H$ normal in G. Then $H=K.$



    We know that Card($G$) = Card($H$) [G:H], so Card($G$) = Card($H$) Card($G/H$). According to one of the isomorphism theorems, $KH<G, (Kcap H) triangleleft H $, and $f: K rightarrow G/H$ is a group homomorphism with ker $ f= Kcap H$ and $K/(K cap H) cong KH/H.$



    Somehow I can not connect the dots. Can somebody help me.
    Many thanks.










    share|cite|improve this question
























      up vote
      2
      down vote

      favorite









      up vote
      2
      down vote

      favorite











      I need to prove the following statement:



      Let $G$ be a finite group, $H, K subset G$ two subgroups, $gcd(Card(H), Card(G/H))=1, Card(H)=Card(K), H$ normal in G. Then $H=K.$



      We know that Card($G$) = Card($H$) [G:H], so Card($G$) = Card($H$) Card($G/H$). According to one of the isomorphism theorems, $KH<G, (Kcap H) triangleleft H $, and $f: K rightarrow G/H$ is a group homomorphism with ker $ f= Kcap H$ and $K/(K cap H) cong KH/H.$



      Somehow I can not connect the dots. Can somebody help me.
      Many thanks.










      share|cite|improve this question













      I need to prove the following statement:



      Let $G$ be a finite group, $H, K subset G$ two subgroups, $gcd(Card(H), Card(G/H))=1, Card(H)=Card(K), H$ normal in G. Then $H=K.$



      We know that Card($G$) = Card($H$) [G:H], so Card($G$) = Card($H$) Card($G/H$). According to one of the isomorphism theorems, $KH<G, (Kcap H) triangleleft H $, and $f: K rightarrow G/H$ is a group homomorphism with ker $ f= Kcap H$ and $K/(K cap H) cong KH/H.$



      Somehow I can not connect the dots. Can somebody help me.
      Many thanks.







      abstract-algebra






      share|cite|improve this question













      share|cite|improve this question











      share|cite|improve this question




      share|cite|improve this question










      asked yesterday









      user249018

      194117




      194117






















          2 Answers
          2






          active

          oldest

          votes

















          up vote
          0
          down vote













          Let $k in K$ and consider $kH in G/H$. Denote $n = |K| = |H|$ and $|k| = m$. Then since the order of $k$ divides the order of $K$, we have $m mid n$.



          Notice that $(kH)^m = H$ in $G/H$, and so $|kH| = r$ must divide $m$ which divides $n$. Since $kH in G/H$, we also have that $r$ divides $|G/H|$. If $r >1$ then we have that $n$ and $|G/H|$ share a common factor greater than $1$, namely $r$. But this is a contradiction since we have assumed $gcd(n, |G/H|) = 1$.



          So it must be that the order of $kH$ is $1$, which then implies that $k in H$. Since $k in K$ was arbitrary we have that $K leq H$. But this implies that $H = K$ since they have the same order.






          share|cite|improve this answer























          • Thanks. I dont understand the implication |$G/H|$ could devide |$K$|.
            – user249018
            yesterday










          • @user249018 Should be fixed now.
            – matt stokes
            yesterday


















          up vote
          0
          down vote













          Matt stokes has already given a good answer, but I think it's a bit indirect, using the orders of elements of the image of $K$ in $G/H$. Instead we can directly consider the size of the image of $K$ in $G/H$ and do the following.



          Let $phi : Gto G/H$ be the quotient map. Consider $phi(K)$. By the first isomorphism theorem, we have $phi(K)cong K/Kcap H$, so $|phi(K)|=|K/Kcap H|$, and $|phi(K)|$ divides $|K|=|H|$, however $phi(K)$ is also a subgroup of $G/H$, so $|phi(K)|$ divides $|G/H|$. Hence $|phi(K)|$ divides $operatorname{gcd}(|H|,|G/H|)=1$. Thus $|phi(K)|=1$, so $Ksubseteq H$, and since $|K|=|H|$, this implies $K=H$.






          share|cite|improve this answer





















            Your Answer





            StackExchange.ifUsing("editor", function () {
            return StackExchange.using("mathjaxEditing", function () {
            StackExchange.MarkdownEditor.creationCallbacks.add(function (editor, postfix) {
            StackExchange.mathjaxEditing.prepareWmdForMathJax(editor, postfix, [["$", "$"], ["\\(","\\)"]]);
            });
            });
            }, "mathjax-editing");

            StackExchange.ready(function() {
            var channelOptions = {
            tags: "".split(" "),
            id: "69"
            };
            initTagRenderer("".split(" "), "".split(" "), channelOptions);

            StackExchange.using("externalEditor", function() {
            // Have to fire editor after snippets, if snippets enabled
            if (StackExchange.settings.snippets.snippetsEnabled) {
            StackExchange.using("snippets", function() {
            createEditor();
            });
            }
            else {
            createEditor();
            }
            });

            function createEditor() {
            StackExchange.prepareEditor({
            heartbeatType: 'answer',
            convertImagesToLinks: true,
            noModals: true,
            showLowRepImageUploadWarning: true,
            reputationToPostImages: 10,
            bindNavPrevention: true,
            postfix: "",
            imageUploader: {
            brandingHtml: "Powered by u003ca class="icon-imgur-white" href="https://imgur.com/"u003eu003c/au003e",
            contentPolicyHtml: "User contributions licensed under u003ca href="https://creativecommons.org/licenses/by-sa/3.0/"u003ecc by-sa 3.0 with attribution requiredu003c/au003e u003ca href="https://stackoverflow.com/legal/content-policy"u003e(content policy)u003c/au003e",
            allowUrls: true
            },
            noCode: true, onDemand: true,
            discardSelector: ".discard-answer"
            ,immediatelyShowMarkdownHelp:true
            });


            }
            });














             

            draft saved


            draft discarded


















            StackExchange.ready(
            function () {
            StackExchange.openid.initPostLogin('.new-post-login', 'https%3a%2f%2fmath.stackexchange.com%2fquestions%2f3005072%2fproving-two-subgroups-with-same-cardinality-are-identical-if-one-is-normal%23new-answer', 'question_page');
            }
            );

            Post as a guest















            Required, but never shown

























            2 Answers
            2






            active

            oldest

            votes








            2 Answers
            2






            active

            oldest

            votes









            active

            oldest

            votes






            active

            oldest

            votes








            up vote
            0
            down vote













            Let $k in K$ and consider $kH in G/H$. Denote $n = |K| = |H|$ and $|k| = m$. Then since the order of $k$ divides the order of $K$, we have $m mid n$.



            Notice that $(kH)^m = H$ in $G/H$, and so $|kH| = r$ must divide $m$ which divides $n$. Since $kH in G/H$, we also have that $r$ divides $|G/H|$. If $r >1$ then we have that $n$ and $|G/H|$ share a common factor greater than $1$, namely $r$. But this is a contradiction since we have assumed $gcd(n, |G/H|) = 1$.



            So it must be that the order of $kH$ is $1$, which then implies that $k in H$. Since $k in K$ was arbitrary we have that $K leq H$. But this implies that $H = K$ since they have the same order.






            share|cite|improve this answer























            • Thanks. I dont understand the implication |$G/H|$ could devide |$K$|.
              – user249018
              yesterday










            • @user249018 Should be fixed now.
              – matt stokes
              yesterday















            up vote
            0
            down vote













            Let $k in K$ and consider $kH in G/H$. Denote $n = |K| = |H|$ and $|k| = m$. Then since the order of $k$ divides the order of $K$, we have $m mid n$.



            Notice that $(kH)^m = H$ in $G/H$, and so $|kH| = r$ must divide $m$ which divides $n$. Since $kH in G/H$, we also have that $r$ divides $|G/H|$. If $r >1$ then we have that $n$ and $|G/H|$ share a common factor greater than $1$, namely $r$. But this is a contradiction since we have assumed $gcd(n, |G/H|) = 1$.



            So it must be that the order of $kH$ is $1$, which then implies that $k in H$. Since $k in K$ was arbitrary we have that $K leq H$. But this implies that $H = K$ since they have the same order.






            share|cite|improve this answer























            • Thanks. I dont understand the implication |$G/H|$ could devide |$K$|.
              – user249018
              yesterday










            • @user249018 Should be fixed now.
              – matt stokes
              yesterday













            up vote
            0
            down vote










            up vote
            0
            down vote









            Let $k in K$ and consider $kH in G/H$. Denote $n = |K| = |H|$ and $|k| = m$. Then since the order of $k$ divides the order of $K$, we have $m mid n$.



            Notice that $(kH)^m = H$ in $G/H$, and so $|kH| = r$ must divide $m$ which divides $n$. Since $kH in G/H$, we also have that $r$ divides $|G/H|$. If $r >1$ then we have that $n$ and $|G/H|$ share a common factor greater than $1$, namely $r$. But this is a contradiction since we have assumed $gcd(n, |G/H|) = 1$.



            So it must be that the order of $kH$ is $1$, which then implies that $k in H$. Since $k in K$ was arbitrary we have that $K leq H$. But this implies that $H = K$ since they have the same order.






            share|cite|improve this answer














            Let $k in K$ and consider $kH in G/H$. Denote $n = |K| = |H|$ and $|k| = m$. Then since the order of $k$ divides the order of $K$, we have $m mid n$.



            Notice that $(kH)^m = H$ in $G/H$, and so $|kH| = r$ must divide $m$ which divides $n$. Since $kH in G/H$, we also have that $r$ divides $|G/H|$. If $r >1$ then we have that $n$ and $|G/H|$ share a common factor greater than $1$, namely $r$. But this is a contradiction since we have assumed $gcd(n, |G/H|) = 1$.



            So it must be that the order of $kH$ is $1$, which then implies that $k in H$. Since $k in K$ was arbitrary we have that $K leq H$. But this implies that $H = K$ since they have the same order.







            share|cite|improve this answer














            share|cite|improve this answer



            share|cite|improve this answer








            edited yesterday

























            answered yesterday









            matt stokes

            524210




            524210












            • Thanks. I dont understand the implication |$G/H|$ could devide |$K$|.
              – user249018
              yesterday










            • @user249018 Should be fixed now.
              – matt stokes
              yesterday


















            • Thanks. I dont understand the implication |$G/H|$ could devide |$K$|.
              – user249018
              yesterday










            • @user249018 Should be fixed now.
              – matt stokes
              yesterday
















            Thanks. I dont understand the implication |$G/H|$ could devide |$K$|.
            – user249018
            yesterday




            Thanks. I dont understand the implication |$G/H|$ could devide |$K$|.
            – user249018
            yesterday












            @user249018 Should be fixed now.
            – matt stokes
            yesterday




            @user249018 Should be fixed now.
            – matt stokes
            yesterday










            up vote
            0
            down vote













            Matt stokes has already given a good answer, but I think it's a bit indirect, using the orders of elements of the image of $K$ in $G/H$. Instead we can directly consider the size of the image of $K$ in $G/H$ and do the following.



            Let $phi : Gto G/H$ be the quotient map. Consider $phi(K)$. By the first isomorphism theorem, we have $phi(K)cong K/Kcap H$, so $|phi(K)|=|K/Kcap H|$, and $|phi(K)|$ divides $|K|=|H|$, however $phi(K)$ is also a subgroup of $G/H$, so $|phi(K)|$ divides $|G/H|$. Hence $|phi(K)|$ divides $operatorname{gcd}(|H|,|G/H|)=1$. Thus $|phi(K)|=1$, so $Ksubseteq H$, and since $|K|=|H|$, this implies $K=H$.






            share|cite|improve this answer

























              up vote
              0
              down vote













              Matt stokes has already given a good answer, but I think it's a bit indirect, using the orders of elements of the image of $K$ in $G/H$. Instead we can directly consider the size of the image of $K$ in $G/H$ and do the following.



              Let $phi : Gto G/H$ be the quotient map. Consider $phi(K)$. By the first isomorphism theorem, we have $phi(K)cong K/Kcap H$, so $|phi(K)|=|K/Kcap H|$, and $|phi(K)|$ divides $|K|=|H|$, however $phi(K)$ is also a subgroup of $G/H$, so $|phi(K)|$ divides $|G/H|$. Hence $|phi(K)|$ divides $operatorname{gcd}(|H|,|G/H|)=1$. Thus $|phi(K)|=1$, so $Ksubseteq H$, and since $|K|=|H|$, this implies $K=H$.






              share|cite|improve this answer























                up vote
                0
                down vote










                up vote
                0
                down vote









                Matt stokes has already given a good answer, but I think it's a bit indirect, using the orders of elements of the image of $K$ in $G/H$. Instead we can directly consider the size of the image of $K$ in $G/H$ and do the following.



                Let $phi : Gto G/H$ be the quotient map. Consider $phi(K)$. By the first isomorphism theorem, we have $phi(K)cong K/Kcap H$, so $|phi(K)|=|K/Kcap H|$, and $|phi(K)|$ divides $|K|=|H|$, however $phi(K)$ is also a subgroup of $G/H$, so $|phi(K)|$ divides $|G/H|$. Hence $|phi(K)|$ divides $operatorname{gcd}(|H|,|G/H|)=1$. Thus $|phi(K)|=1$, so $Ksubseteq H$, and since $|K|=|H|$, this implies $K=H$.






                share|cite|improve this answer












                Matt stokes has already given a good answer, but I think it's a bit indirect, using the orders of elements of the image of $K$ in $G/H$. Instead we can directly consider the size of the image of $K$ in $G/H$ and do the following.



                Let $phi : Gto G/H$ be the quotient map. Consider $phi(K)$. By the first isomorphism theorem, we have $phi(K)cong K/Kcap H$, so $|phi(K)|=|K/Kcap H|$, and $|phi(K)|$ divides $|K|=|H|$, however $phi(K)$ is also a subgroup of $G/H$, so $|phi(K)|$ divides $|G/H|$. Hence $|phi(K)|$ divides $operatorname{gcd}(|H|,|G/H|)=1$. Thus $|phi(K)|=1$, so $Ksubseteq H$, and since $|K|=|H|$, this implies $K=H$.







                share|cite|improve this answer












                share|cite|improve this answer



                share|cite|improve this answer










                answered yesterday









                jgon

                9,69211538




                9,69211538






























                     

                    draft saved


                    draft discarded



















































                     


                    draft saved


                    draft discarded














                    StackExchange.ready(
                    function () {
                    StackExchange.openid.initPostLogin('.new-post-login', 'https%3a%2f%2fmath.stackexchange.com%2fquestions%2f3005072%2fproving-two-subgroups-with-same-cardinality-are-identical-if-one-is-normal%23new-answer', 'question_page');
                    }
                    );

                    Post as a guest















                    Required, but never shown





















































                    Required, but never shown














                    Required, but never shown












                    Required, but never shown







                    Required, but never shown

































                    Required, but never shown














                    Required, but never shown












                    Required, but never shown







                    Required, but never shown







                    Popular posts from this blog

                    Can a sorcerer learn a 5th-level spell early by creating spell slots using the Font of Magic feature?

                    ts Property 'filter' does not exist on type '{}'

                    mat-slide-toggle shouldn't change it's state when I click cancel in confirmation window